You are on page 1of 105

Dr.

Sameir Abd Alkhalik Aziez


University of Technology
Department of Electromechanical Engineering

Basics of Electrical Circuits


Resources : Introductory circuit Analysis; by Robert L. Boylestad .
Basic Electrical Engineering science; by Mckenzie smith and K.T. Hosie
Electrical Technology.
. & .
Definitions :Electric charge : Fundamental property of sub-atomic particles .
Positive charge

proton

negative charge

electron

no charge

neutron

Like charges repel & opposite charges attract.


Unit of charge
Symbol

coulomb

and

Units:-

Difference

between

symbol

and

unit.

( Symbol ) = 180C (Unit ) .

Current :- The movement of electrons is the current which results in work


begin done in an electric circuit. Hence, the Electric Current is the rate of flow
of charge.
We have a current when there is a few of charges;
I=

q
C
A=
t
S

The direction of motion of positive charges is opposite to direction of motion of


negative charges.
Amper =

Coul. 1.6 1019 106 q


=
=
=I
t
Sec.
t

q = Charge that flows through section in time t.

Uniform flow of charges direct current ( dc )

-1-

Example: In a copper wire a flow of charge is 0.12 C in a time of 58 ms . Find


the current in this wire ?

Solution: I =

q
t

0.12
58 10 3

= 2.06 A

Work & Energy:


w = F .d

Where:
w : Is the work done by a force ( F ) acting over a displacement ( d ).

As charges move, subject to various forces they may gain or lose energy.
w = work done = charge in energy

External force

work done on charges

charges gain energy.


( Like Sources )

Work done by charges

charges lose energy


(Like loads )

w = Joules = N.m

Potential difference:
Potential difference between two points is the energy gained or lost by a unit
charge as it moves from on point to the other.
v=

w
Joule
volt =
q
coulomb

w = work done for transporting a total charge q .

Potential difference
Voltage
Potential

-2-

Power :- Time rate for doing work .


P=

w
t

P=

w q w q

= V .I
t q q t

Watt (w) =

J
= V.A
s

Systems of unit ( S.I. ) :Quantity

Unit

Symbol

1) Charge (q)

Coulombs

2) Current (I)

Amper

A=

3) Force (F)

Newton

4) Work , energy (w)

Joule

J=N.m

5) Voltage (V)

Volt

V =

J
c

6) Power (P)

watt

w=

J
= V .A
s

7) Length ( l )

meter

8) Temperature (T)

Kelvien

-3-

C
S

Electrical Circuits :Circuit element :- is a two terminal electrical component .


Electrical circuit :- Interconnected group of elements .
+

+-

VA

VA
VA

+
I

I
V

VB

(1)
Load
(dissipates power)
P = V.I

VB
VB

(2)
Source
(supplies power)

I
V

I
+-

VA

I
V

I
-

VA

VB

(3)
Source
(supplies power ,
charging)

VB

(4)
Load
(dissipates power)

Source = current and voltage in the same direction.


load = current and voltage in the opposite direction .
Resistive element, is always load and always dissipates power

+
I

-4-

Source of emf ( electro motive force )


V

Source ( supplies power )

Load ( consumes power )

Ex. Charging of batteries.

Resistance :The unit of resistance ( R ) is ohm ( )

The resistance of any material is depends on four factors:


1. The length of conductor ( l ) R l
2. Cross section area of conductor ( A ) R

1
A

3. The nature of material conductor.


4. The temperature of conductor.
l

l
A

R=

l
A

= constant is called resistivity or specific resistance unit of resistivity.

-5-

RA .m 2
l
R= =
=
= .m or .cm
l
m
A
d
A = r =
2

r = radians of section.
d = diameter of section.
Conductance ( G ):G =

1
=
R

A
l

(Siemens (S))

Prefix :pico

10-12

nano

10-9

micro

10-6

milli

10-3

centi

10-2

deci

10-1

Kilo

103

Mega

106

Giga

109

Tera

1012

Example: What is the resistance of 3 Km length of wire with cross section area
6 mm2 and resistivity 1.8 cm .
Solution:

R=

l
A

(1 .8 10
=

) (
)

10 2 3 10 3
= 9
6 10 6

-6-

Example: What is the resistance of 100 m length of copper wire with a


diameter of (1 mm) and resistivity 0.0159 m .
Solution:
R =

l
A

(0 . 0159

10

) 100

d

2

(0 . 0159

10

1 10

) 100
2

= 2 . 02

Effect of Temperature on a resistance :-

slop =

R R
R R R R1
R
=
= constant = 2 1 = 2
T2 T1
T2 T
T T1
T

Example: The resistance of material is 300 at 10Co, and 400 at 60Co. Find
its resistance at 50 Co ?
Solution:
slop =
2=

R2 R1 400 300
=
= 2 / C o
T2 T1
60 10

R R1 R 300 R 300
=
=
50 10
40
T T1

R 300 = 80 R = 80 + 300 R = 380


-7-

Also from the above figure we can sea


R2 0 R1 0
=
T2 T0 T1 T0
R2
R1
=
T2 T0 T1 T0

R2 T2 T0
=
R1 T1 T0

l
A = T2 T0 2 = T2 T0
, hence
l
T1 T0
1 T1 T0
1
A

Example: Aluminum conductor has resistance 0.25 at 10 Co . Find its


resistance at 65 Co ?
Solution:
T T0
R 2 T2 T0
=
R 2 = R1 2

T1 T0
R1
T1 T0
65 ( 236 )
R 2 = 0 .25

10 ( 236 )
301
= 0 . 25
= 0 .31 .
246

The following table illustrate the value of resistivity ( ) for different materials
at 20 Co temperature.
Material

at 20 C , .m

Material

To , Co

Silver

1.63*10-8

Silver

-234

*Copper

(1.72 - 1.77)*10-8

Copper

-234.5

Aluminum

2.83*10-8

Aluminum

-236

Iron

20*10-8

Iron

-180

Paper

10*1010

Mica

1011-1015
-8-

Example: Aluminum conductor with length of 75 cm and 1.5 mm2 cross


section area. Find its resistance at 90 Co ?
Solution:

R=

l
A

(2.83 10 ) (75 10 )
=
8

1.5 10 6

= 2.83 50 10 4 = 141.5 10 4 = 14.15m


. 20Co :
R2
T T0
= 2
T1 T 0
R1
90 ( 236 )
R 2 = 14 . 15

20 ( 236 )
90 + 236
= 14 . 15
= 18 m .
20 + 236

Another method

2 T2 T0
90 + 236
=
90 = 20

1 T1 T0
20 + 236

R=

90 l
A

90 + 236
2
75 10
20 + 236
1.5 10 6

20

20 = 2.83 10 8
90 = 14.15m

-9-

Driving :
R2
T T0
= 2
R1
T1 T0
T
R 2 = R1 2
T1
T
R 2 = R1 2
T1

T0
T0

T0
+ 1 1
T0

T T0
R 2 = R1 1 + 2
1
T1 T0

T T0 (T1 T0 )

R 2 = R1 1 + 2

T
T
1
0

T T1

R 2 = R1 1 + 2

T
T
1
0

1
(T2 T1 )
R 2 = R1 1 +

T1 T0

Let 1 =

1
T1 T0

temperature coefficient of resistance at a temperature T1

R2 = R1[1 + 1 (T2 T1 )]

Where T0 for copper = -234.5


In some resource, T0 take an absolute value, which means T0 = 234.5, hence
we can sea
1 =

1
T + T1

&

T0 + T2
R2
=
R1
T 0 + T1

Example:
a) Find the value of 1 at (T1 = 40 Co) for copper wire.
b) Using the result of (a), find the resistance of a copper wire at 75 Co if its
resistance is 30 at 40 Co ?
-10-

Solution:
a)

Or

1 =

1 =

1
1
1
=
=
= 0.00364
T1 T0 40 ( 234.5) 274.5

1
1
1
=
=
= 0.00364
T + T1 234.5 + 40 274.5

1/K

1/K

R2 = R1[1 + 1 (T2 T1 )]

b)

= 30[1 + 0.00364(75 40)] = 33.8

.
Example: If the resistance of a copper wire at freezing ( 0 Co ) is 30 , Find its
resistance at -40 Co ?
Solution:
T T0
R2 = R1 2
T1 T0

40 ( 234.5)
= 30

0 ( 234.5)

194.5
= 30
= 24.88
234.5

Or

R 2 T0 + T2
=
R1 T0 + T1

234.5 40
= 30

234.5 + 0
194.5
= 30
= 24.88
234.5

-11-

Ohm's Law :- Ohm's law states that the voltages ( V ) across a resistor ( R ) is
directly proportional to the current ( I ) flowing through the resistor .

Slop =

I
1
=
V R

V
= constant = R
I

R=

V
I

; V=I.R ;

I=

V
R

The resistance of short circuit element is approaching to zero.


The resistance of open circuit is approaching to infinity.

O.C.

Hence G =

R=
1
G= =0
R

1 I
=
R V

S.C.

Siemens ( S ) or mhos ( ) .

-12-

R=0
1
G= =
R

Electrical Energy ( W ) :Q P =

W
t

= P .t

KWh

W = P .t
= (V . I ).t

V 2
=
R

.t

= I 2 . R .t

Energy in KWh ( W ) =

Power (P ) time(t )
1000

Example : For the following circuit diagram , calculate the conductance and
the power ?

Solution :
I=
G=

V
30
=
= 6mA
R 5 103
1
1
=
= 0.2m
R 5 103

P = I .V = 6 103 30 = 180mW

or P = I 2 .R = (6 103 ) (5 103 ) = 180mW


or P = V 2 .G = (30)2 (0.2 103 ) = 180mW

(30) = 180mW
V2
=
R
0.2 10 3
2

or P =

-13-

Efficiency ( ) :-

Wi/p = Wo/p + Wloss


Wi / p
t

Wo / p
t

Wloss
t

Pi/p = Po/p + Ploss


Efficiency ( ) =
=

Po
100%
Pi

Wo
100%
Wi

= 1

Output power
100%
Input power

......

Example: A 2 hp motor operates at an efficiency of 75 %, what is the power


input in Watt, if the input current is (9.05) A, calculate also the input
voltage?
Solution:
1 hours power (hp) = 746 Watt
=

Po
100%
Pi
-14-

0.75 =

2 746
1492
Pi =
= 1989.33W
Pi
0.75

P = E.I E =

P 1989.33
=
= 219.82 220V
9.05
I

Example: What is the energy in KWh of using the following loads:a) 1200 W toaster for 30 min.
b) Six 50 W bulbs for 4 h.
c) 400 W washing machines for 45 min.
d) 4800 W electric clothes dryer for 20 min.
Solution :
W =

P(W ) t (h )
1000

30
45
20
1200 + 6 50 4 + 400 + 4800
60
60
60
W =
1000
=

600 + 1200 + 300 + 1600 3700


=
= 3.7 KWh
1000
1000

D.C. Sources:The d.c. sources can be classified to:1- Batteries .

Voltage
Amper - hours

2- generators .
3- Photo cells .
4- Rectifiers .

-15-

V
E

V
E

V = E = constant voltage element

V
Io
I
Io

I = Io = constant current element


.
.

Series Circuit :-

V1 = I.R1
V2 = I.R2
V3 = I.R3
E V1 V2 V3 = 0 E = V1 + V2 + V3
E = I.R1 + I.R2 + I.R3

-16-

E = I.[R1 + R2 + R3] = I.RT


The current in the series circuit is the same through each series element &
RT = R1 + R2 + R3 + -------- + RN

I=

E V1 V2 V3
=
=
=
RT R1 R2 R3

Pt = P1 + P2 + P3 = E.I
Voltage Source in Series:-

-17-

Example: Find the current for the following circuit diagram?

Solution:
ET = 10 + 7 + 6 3 = 20 V
RT = 2 + 3 = 5
I = IT =

ET 20
=
= 4A
RT
5

Kirchoff's voltage law ( K.V.L. ):The algebraic sum of all voltages around any closed path is zero.
m

m =1

Vm = 0

Where m is the number of voltages in the path ( loop ) , and Vm is the mth
voltage .

EV=0
E=V
I=

V E
=
R R

-18-

E V1 V2 = 0
E = V1 + V2

I =

; RT = R1 + R2

E
V + V2
= 1
RT
RT

Example: Use K.V.L. to find the current in the following circuit diagram?
I

R1
V1
E2

E1
V2
R2

Solution:

From K.V.L.

V = 0

E1 V1 E2 V2 = 0

E1 E2 = V1 + V2
E1 E2 = IR1 + IR2
E1 E2 = I ( R1 + R2 )
I =

E1 E2
R1 + R2

-19-

Example: For the following circuit diagram, Find I using:a) Ohm's law.
b) K.V.L.
7

40V

10
20V

10V
17

Solution:
a ) By applying ohm's law :-

I=

ET 20 + 40 10 10 40
=
=
= 1A
R
10 + 7 + 6 + 17
40

b ) By applying K.V.L. :10 + 6I + 7I - 40 + 10I 20 + 10 + 17I = 0


10 40 20 + 10 + I ( 6 + 7 + 10 + 17 ) = 0
-40 = -I ( 40 ) I =

40
= 1A
40

-20-

10V

Example :- For the following circuit diagram , find the potential difference
between Node ( A & D ) , and Node ( A & F ) ?

D
12V

6V

8V
B

E
5V

Solution : To find the potential difference between Node A & D , we will apply
K.V.L. on the closed loop BADEB

+6 + V 12 8 = 0

V = 20 6 = 14 volt
A

or
+6 V1 12 8 = 0

V2

V
V1

D
12V

6V

8V

14 V1 = 0
B

V1 = 14 volt

E
5V

D *
. 14 A
C

Take the loop FEDAF to find the potential difference between Node C & F .
5 + 12 V V2 = 0

5 + 12 14 V2 = 0 7V2 = 0

V2 = 7 volt .

Or Take the loop FEBAF 5 8 +6 V2 = 0 V2 = 7 volt .

-21-

Example :- For the following circuit diagram , find the current ?

Solution :

A
2

2
2

I
8V

15V

Take the loop FABCDEF


+8 + V 15 = 0 +V 7 = 0 V = +7 volt
V = IR I =

7
= 3.5 A
2

Definitions :Node :- Meeting point of 3 or more branches .


Branch :- Series of elements carrying the same current .
Loop :- Is any closed path in a circuit .

-22-

Hence for the loop circuit, we can find :A

4 nodes and 6 branches


E1

and we can find : V1 , V2 , V3 and V4

V2

as follows :-

Take the loop BACB ; to find V1


E2 V1 E3 = 0 V1 = E2 E3

E2

V3

V1

E3

V4

Or, if we take BCAB ;


E3 V1 E2 = 0 V1 = E2 E3

Take the loop ADBA ; to find V2


E1 + V2 + E2 = 0 V2 = E1 E2
Take the loop ABCDA ; to find V3
E2 + E3 + V3 + E1 = 0
V3 = E2 E3 E1
V4 = V3
Or;
E2 + E3 V4 + E1 = 0
V4 = E1 + E3 E2

Example :- For the following circuit diagram , find ; RT , I , V1 , V2 , P4 , P6


, PE , verify by K.V.L. ?
R1
I
E=20V

Solution :RT = R1 + R2 = 4 + 6 = 10
I=

E
20
=
= 2A
RT 10
-23-

4
V1

R2
6
V2

V1 = IR1 = 2 4 = 8V
V2 = IR2 = 2 6 = 12V

; or

V12 (8)
=
= 16w
P4 =
R1
4

P6 = I R2 = (2 ) 6 = 24W

; or

V 2 (12 )
= 24w
P6 = 2 =
R2
6

PE = IE = 2 20 = 40W

; or PE = P4 + P6 = 16 + 24 = 40W

P4 = I 2 R1 = (2) 4 = 16W
2

To verify results by using K.V.L. ; then


N

V
i =1

=0

E V1 V2 = 0
E = V1 + V2
20 = 8 + 12
20 = 20

checks

Internal Resistance :Every practical voltage or current source has an internal resistance that
adversely affects the operation of the source.
In a practical voltage source the internal resistance represent as a resistor in series
with an ideal voltage source.
In a practical current source the internal resistance represent as a resistor in
parallel with an ideal current source, as shown in the following figures.

I
Ro

Vo

RL

Ro

Practical voltage source

Practical current source

-24-

RL

Where
Ro = Internal resistance
RL = load resistance
According to K.V.L.
E Vo V = 0
E IRo V = 0
V = E IRo

Note that an ideal sources have Ro = 0


We can representing a load as a group of parallel resistances.
Hence as the load will increase the current will be increase ( because the
resistance will decrease ) and the voltage will decrease .

This is because the drop voltage due to the internal resistance , as shown in the
following figure :-

-25-

As seen from the above figure , if Ro2 > Ro1 , then V2 < V1 and the drop voltage
will be ( E V2 ) , which is greater than ( E V1 ) .
Example :- For the following circuit diagram , calculate I and VL for the
following cases :a) Ro = 0
b) Ro = 8
c) Ro = 16

Solution :a.) By apply K.V.L.


E Vo V = 0
120 IRo IRL = 0

120 0 22I = 0

120 = 22I I = 5.46 A


VL = I RL = 5.46 * 22 = 120 V
b.) E Vo V = 0
120 8I 22I = 0

120 30I = 0

120 = 30I I = 4 A
VL = I RL = 4 * 22 = 88 V
c.) E Vo V = 0
120 16I 22I = 0

120 38I = 0

120 = 38I I = 3.16 A


VL = I RL = 3.16 * 22 = 69.5 V
Then we can conclude that as Ro increase the total current and load voltage will
decrease.
-26-

Example :- A circuit have load one with 20 and 4A , and load two with 10
& 6A . Find the current for load three which have 30 ?
Solution :1) 20 & 4A
2) 10 & 6A
3) 30 & I = ?
From K.V.L. , then
E Vo VL = 0
VL = E IRo
IRL = E IRo
4 * 20 = E 4Ro
80 = E 4Ro

-----------------

(1)

-----------------

(2)

Also
6 * 10 = E 6 Ro
60 = E 6Ro
From eq. (1) & (2) , we have
20 = ( 6 4 ) Ro Ro = 10 ; sub. this result it in eq. (1) , then
80 = E 4 * 10 E = 120 V
Now , we Apply K.V.L. for load 3 ;
V3 = E IRo 30I = 120 10I
40I = 120 I = 3 A for load three.
See from this example that the current will increase as the load will decrease
with constant E & Ro .

-27-

Example :- A circuit have Voc = 25 v and Isc = 50 A , find its current and RL
when VL = 15 V ?

Solution :E = Voc = 25 V
Ro =

E 25
=
= 0.5
I sc 50

From K.V.L.
E Vo VL = 0
E 0.5I 15 = 0
25 0.5I 15 = 0
0.5I = 25 15

I =
RL =

10
= 20 A
0.5

VL 15
=
= 0.75
20
I

-28-

Voltage divider Rule :RT = R1 + R2


I=

E
RT

E
E.R1
V1 = I .R1 = .R1 =
RT
RT
E
E.R2
V2 = I .R2 = .R2 =
RT
R2

Vn =

ERn
RT

Voltage divider rule

Vn = Voltage across Rn
E = The ( emf ) voltage across the series elements .
RT = The total resistance of the series circuits .
Example :- Using voltage divider rule , determine the voltage V1 , V2 , V3 and
V4 for the series circuit in figure below , given that ; R1 = 2K , R2 = 5K ,
R3 = 8K , E = 45 V ?
Solution :V4
I

V1 =

R1 E 2 *10 3 * 45
=
= 6V
RT
15 *10 3

R2 E 5 *103 * 45
V2 =
=
= 15V
RT
15 *103

-29-

R1

R2

R3

V1

V2

V3

V3 =

V4

R3 E 8 *103 * 45
=
= 24V
RT
15 *103

(R + R2 )E
= 1
RT

To check:

7 *103 * 45
=
= 21V
15 * 103

or V4 = V1 + V2 = 21V

E V 1 V2 V3 = 0
E = V1 + V2 + V3 45 = 6 + 15 + 24
45 = 45

Active Potential :

Va = 14 V a

Vab

is the voltage difference between the

Vab = 6 V

point a and point b


Vab = Va Vb = 14 8 = 6V

Vb = 8 V

Vba = Vb Va = 8 14 = - 6V
Vab = - Vba
a

Va = -7 V

-7 V

Va = 10 V

10 V

Va = 0 V

-30-

E = 20 V

R1

R1

20 V

R2

R2

E = -12 V

R1

R1

-12 V

R2

R2

-31-

Example :- Find Va , Vb , Vc , Vab , Vac and Vbc for the following diagram .

Solution :RT = R1 + R2 + R3
= 2 + 5 + 3 = 10
I=

E 10
=
= 1A
RT 10

E V2 Va = 0
Va = E V2 = 10 (2 * 1) = 8 V
Vb = V5 = (1 * 5) = 5 V = Vbc
or

E V 2 V3 Vb = 0

; Vc = 0 V
Vb = E V2 V3 = 10 2 3 = 5 V

Vab = Va Vb = 8 5 = 3 V
Vac = Va Vc = 8 0 = 8 V
Vbc = Vb Vc = 5 0 = 5 V

-32-

Equivalence of actual sources :-

Open

Voc = E

Circuit

I=0

Short

I sc =

circuit

Voltage

Current

Source

Source
Voc = I o

E
Ro

1
Go

Isc = Io

V=0

Kirchoff's Current Law ( K.C.L. ) :The algebraic sum of ingoing currents is equal to the out going currents
at any point .

in

= I out

Or , At any point , the algebraic sum of entering and leaving current is zero .

I = 0
I2

I3

I1
I4
I5

I1 + I2 + I4 = I3 + I5
Or

I 1 + I2 + I4 - I 3 - I 5 = 0

-33-

At a
Or

I1 = I2 + I3

13 + 5 I = 0

I1 - I 2 - I 3 = 0

18 I = 0

At b

I = 18 A

-I1 + I2 + I3 = 0
Example :- Find the current in each section in the cct. Shown ?
1A
a
Iab
3A

b 3A

e
2A

Ibc
Ide

c
4A

8A
Icd

Solution :At node a


3 1 Iab = 0
2 Iab = 0 Iab = 2 A

-34-

At node b
Iab + 3 Ibc = 0
2 + 3 Ibc = 0 Ibc = 5 A
At node c
Ibc + 4 Icd = 0
5 + 4 Icd = 0 Icb = 9 A
At node d
Icd 8 Ide = 0
9 8 Ide = 0 Ide = 1 A
At node e
2 3 + Ide = 0
23+1=0
0 = 0 check .
Example :- Find the magnitude and direction of the currents I3 , I4 , I6 , I7 in the
following cct. Diagram?
b

I2

8A

12

I5

I1 = 10A a

I4

I7
I6

I3
c

Solution :-

enter

= I leave

I1 = I7 = 10 A

-35-

At node a ; suppose I3 is entering


I1 + I3 I 2 = 0
10 + I3 12 = 0 I3 = 2 A
At node b;
I2 enter , I5 leave , I4 must be leaving
I2 = I5 + I4
12 = 8 + I4 I4 = 12 8 = 4 A
At node c;
I4 enter , I3 leave , I6 leave
I4 = I3 + I6
4 = 2 + I6 I6 = 2 A
At node d;
I5 and I6 enter , I7 leave
I7 = I5 + I6
10 = 8 + 2
10 = 10

Ok.

Resisters in Parallel :-

I
I2

I1
V

V1

R1

V2

From K.V.L.

V = V1 = V2

From K.C.L.

I = I 1 + I2

R2

I=

From .L.
B

V1 V2
+
R1 R2

= V1G1 + V2G2
= V1 ( G1 + G2 )
or

I = V ( G1 + G2 )
I = VGT

-36-

Where

GT = G1 + G2
R + R2
1
1
1
=
+
= 1
RT
R1
R2
R 1 .R 2

Hence

or

RT =

R1.R2
R1 + R2

In the same minner , if we have three resistors in parallel , then:


1
1
1
1
=
+
+
RT
R1
R2
R3
1
R .R + R1 .R 3 + R1 .R 2
= 2 3
RT
R1 .R 2 .R 3
RT =

R1 .R 2 .R 3
R 2 .R 3 + R1 .R 3 + R1 .R 2

And , if we have N of parallel


resistance , then
1
1
1
1
1
=
+
+
+
RT
R1 R 2 R 3
RN

Also
PT = P1 + P 2 + P3
V12
P1 = V1 I 1 = I R1 =
R1
2
1

Source power Ps = EI T = I T2 R T =

E T2
RT

-37-

Example :- For the following cct. Find RT , PT , IT , Ib?


IT

16 V

Solution :RT =
IT =

R 8
= = 2
N 4

E 16
=
= 8A
RT
2

I branch =

E 16
=
= 2A
R1
8

PT = IT2 RT = (8) .(2) = 128W


2

or

PT = E.IT = 16 * 8 = 128W

or

P T = P 1 + P 2 + P 3 + P4
= (2 ) * 8 + (2 ) * 8 + (2 ) * 8 + (2 ) * 8
2

= 32 + 32 + 32 + 32 = 128W

-38-

Example :- For the parallel network in fig. below , find :a) R3 , b) E , c) IT , I2 , d) P2 ; given that RT = 4 ?

Solution :a)
1
1
1
1
=
+
+
RT R1 R2 R3
1 1
1
1
=
+
+
4 10 20 R3
0.25 = 0.1 + 0.05 +

1
R3

0.25 0.1 0.05 =

1
R3

0.1 =

b)
c)

1
1
R3 =
= 10
0.1
R3

E = V1 = I1R1 = 4 * 10 = 40 V

IT =

E 40
=
= 10 A
RT
4

I2 =

V2
E 40
=
=
= 2A
R2 R2 20

d)

P2 = I 22 R2 = (2) .(20) = 80W

or

P2 =

V22
R2

, or P2 = I2V2

-39-

Current division Rule :V =I

I=

R1 .R2
R1 + R2

V
=
RT

I1 = I

R1.R2
R1 + R2
R1

R2
R1 + R2

In the same miner


I2 = I

R1
R1 + R2

R2
I
R
R +R
Also 1 = 1 2 = 2
I 2 I R1
R1
R1 + R2
I

I1 R2 G1
=
=
I 2 R1 G2

Example :- For the following circut. , find V , I1 and I2?

Solution :RT =

100 * 0.1
10
R1.R2
=
=
= 0.0999
R1 + R2 100 + 0.1 100.1

V = I . RT = 5 * 0.0999 = 0.4995 V

-40-

I1 =

V
= 0.004995 A
100

I2 =

V
= 4.995 A
0.1

To check

I = I 1 + I2
5 = 0.004995 + 4.995
5 = 5 Ok.

Example :- Determine the resistance R1 in the figure below?

Solution :I = I1 + I2
or

I2 = I I1 = 27 21 = 6 mA
V2 = I2R2 = 6 * 10-3 * 7 = 42 mV
V1 = V2 = 42 mV
R1 =

V1 42 *10 3
=
= 2
I 1 21 *10 3

or
I1 = I

27 *103 * 7
R2
21 *10 3 =
R1 + R2
R1 + 7

R1 = 2

-41-

Voltage Regulation :Voltage Regulation

VR % =

VNL VFL
100%
VFL

Where
VNL = No load voltage
VFL = Full load voltage
Also we can write
VR % =

R int .
100 %
RL

Where
Rint. = Internal resistor .
RL = load resistor .
Example :- Find the voltage VL and power lost to the internal resistance , if the
applied load is 13 , also find the voltage regulation ?
Rint. = 2

VL

E = 30V

RL = 13

Solution :IL =

E
30
=
= 2A
Rint + RL 2 + 13

VL = E I L Rint . = 30 2 * 2 = 26V
Ploss = I L2 Rint . = (2 ) .(2 ) = 8W
2

or

VR % =

30 26
VNL VFL
100% =
100% = 15.385%
26
VFL

VR % =

2
Rint .
100% = 100% = 15.385%
13
RL

-42-

Example :- Find the current I , for the network shown:


I = 42 mA

I1
R2 =24

R1 = 6

R3 =24

Solution :- All resistance in parallel , so if we define that R = R2 // R3 then :R=

24 * 24
R2 R3
=
= 12
R2 + R3 24 + 24

Hence
I1 = I

R
12
= 42 * 10 3
= 28mA
R + R1
12 + 6

Example :- Calculate I & V for the network shown

Solution :- We have a short circuit on R2 resistance , hence no current through


R2 , hence the above cct. Can redrawn as fellows:

-43-

I=

E
E
18
=
=
= 3.6mA
RT R1 5 * 103

V = I .R1 = E = 18V

Example :- For the following cct. Network , find RT , IA , IB , IC , VA , VB , I1 ,


I2 ?
I1 R1 = 9
IA
I2

IB
R2 = 6

IC
R3 = 4
R6 = 3

16.8 V
R4 = 6

R5 = 3

Solution :-

RA =

9*6
R1R2
=
= 3.6
R1 + R2 9 + 6

RB = R3 + R4 // R5 = 4 +

IA

RB = 6

RT = RA + RB // RC

IA =

IC
IB

9*3
= 6
6+3

RC = 3

= 3.6 +

RA = 3.6

6*3
= 5.6
6+3

E 16.8
=
= 3A
RT
5.6
-44-

RC = 3

Apply C.D.R.
IB =

3*3
I A RC
=
= 1A
RB + RC 3 + 6

By K.C.L.
IC = IA I B = 3 1 = 2 A
VA = IARA = 3 * 3.6 = 10.8 V
VB = IBRB = 1 * 6 = 6 V = VC

I1 =

6*3
I A R2
=
= 1.2 A
R1 + R2 6 + 9

I2 = IA I1 = 3 1.2 = 1.8 A
To check
E VA VB = 0
16.8 10.8 6 = 0
0=0

Ok.

-45-

Example :- Find the resistor required to connect in parallel with the ammeter to
flow 1.2 A , if you know that the fsd ( full scale deflection ) of ammeter is 120
mA , and the resistance of ammeter is 2.7 ?
Solution :From K.C.L.
Ish = 1.2 0.12 = 1.08 A
I sh = I

RA
RA + Rsh

2.7

1.08 = 1.2
2.7 + Rsh

1.08 =

3.24
2.7 + Rsh

3.24 = 1.08(2.7 + Rsh )


3.24 = 2.916 + 1.08Rsh
0.324 = 1.08Rsh
Rsh = 0.3

-46-

Example :- for the following cct. Network , Given that (V= 24 v), Find E ?
4

Solution :I1 =

24
= 2A
1.2

16

12

The same voltage ( V = 24 V ) on the


4

resistor Ra = 4 +16 + 4 = 24
Hence

24
I2 =
= 1A
24

24

I 3 = I1 + I 2 = 3 A

Also from K.C.L. I4 = I3 = 3A


Take the closed loop ABCDA , from
8

K.V.L.
V1 6I4 V 6I3 = 0

I3

V1 = 6 * 3 + 24 + 6 * 3

I2

I1
16

12

60
= 2.5 A
24

D
I4

I 6 = I 5 + I 4 = 2.5 + 3 = 5.5 A

Take the closed loop CBC

V1

V1 V2 + E V3 = 0
E = V1 + V2 + V3

V1 = 60V

I5 =

24

I5
C

E = 60 + 25.5 * 8 + 5.5 * 8
V2

= 60 + 44 + 44
E = 148 V

-47-

V3

I6

Current Source :Example :- Find the voltage ( Vs ) for the circuit below:
10 A

Solution :Vs = IRL = 10 * 2 = 20 V

if RL = 2

Vs = IRL = 10 * 5 = 50 V

if RL = 5

RL (2-5)

Vs

Example :- Calculate V1 , V2 , Vs for the following cct.:


Solution :V1 = IR1 = 5 * 2 = 20 V
V2 = IR2 = 5 * 3 = 15 V

I = 5A

Vs = V1 + V2 = 10 + 15 = 25 V

V1

R1

V2

R2

Vs

Source Conversions :A voltage source with voltage E and series resistor Rs can be replaced by
a current source with a current I and parallel resistor Rs as shown :-

I=

E
Rs

Current source to voltage source


Voltage source to current source

-48-

Example :- Convert the voltage source in the cct. Below to a current source ,
then calculate the current through the load for each source:
IL

Solution :Rs = 2

E
6
IL =
=
= 1A
Rs + RL 2 + 4

RL = 4

E = 6V

For the current source cct.

Rs
2
IL = I
= 3
= 1A
Rs + RL
2+ 4

3A

IL
I=

E
Rs

IL

Rs = 2

6
= = 3A
2

RL = 4

Example :- Convert the current source in the cct. Shown below to a voltage
source and determine IL for each cct.:

Solution : For the current cct.


IL = I

Rs
= 9 * 10 3
Rs + RL

) 3 *103 *+106 *10


3

I L = 3mA

For the voltage source cct.


IL =

E
E
27
=
=
RT Rs + R L (3 + 6) *10 3

I L = 3mA

-49-

Current source in parallel :-

Is = 10 6 = 4 A &

Rs = 3 // 6 = 2

Example :-

Is = 7 3 + 4 = 8 A
Example :- Find the load current in the following cct.:

Solution :-

-50-

I1 =

E 32
=
= 4A
R1
8

Is = I1 + I2 = 4 + 6 = 10 A
Rs = R1 // R2 =

8 * 24
Rs
10 * 6
= 6 I L = I s
=
= 3A
8 + 24
Rs + RL 6 + 14

-51-

Matrices :Second order determinate

D=

Col. 1

Col. 2

a1

b1

a2

b2

= a1b2 a2b1

Col. 1

Col. 2

a1x

b1y

c1

a2x

b2y

c2

a1
a
2

Col. 3

b1 x c1
=
b2 y c2

c1 b1

D1 c2 b2 c1b2 b1c2
x=
=
=
D a1 b1 a1b2 b1a2
a b
2 2
a1

D2 a2
y=
=
D a1
a
2

c1
c2 a1c2 c1a2
=
b1 a1b2 b1a2
b2

Example :- Find the value of D


4 1
D=

6 2

Solution :4 1
D=
= 4 * 2 ( 1) * 6 = 8 + 6 = 14
6 2

-51-

Example :- Solving the equations below by determinates


4I1 6 I2 = 8
2I1 + 4 I2 = 20

Solution : 4 6 I1 8
2 4 I = 20

8 6

D1 20 4 8 * 4 ( 6 ) * 20 32 + 120
I1 =
=
=
=
= 5.428 A
D
4 * 4 ( 6) * 2
16 + 12
4 6
2 4

4 8
2 20
D
= 4 * 20 8 * 2 = 2.28 A
I1 = 1 =
D
28
28

Third order determinant :-

a1
D = a2

b1
b2

c1 a1 b1
c2 a2 b2

a3

b3

c3 a3 b3

D = [a1b2 c 3 + a1c 2 a 3 + c1 a 2 b3 ] [c1b2 a 3 + a1c 2 b3 + b1 a 2 c 3 ]

Example :- Find the value of D


1 2 3
D = 2 1 0
0 4 2

-52-

Solution : 1 2 3 1 2
D = 2 1 0 2 1
0 4 2 0 4
D = [1 * 1 * 2 + 2 * 0 * 0 + 3 * ( 2 ) * 4] [0 * 1 * 3 + 4 * 0 *1 + 2 * 2 * 2]
D = [2 + 0 24] [0 + 0 + 8] = 22 + 8 = 14

Example :- Find V1 , V2 , V3 from the following equations :2V1 + 4V2 +2V3 = 8


5V1 2V2 10V3 = 18
V1 + 8V2 20V3 = -8
Solution :2 V1 8
2 4
5 2 10 V = 18
2

1 8 20 V3 8

4
2 8 4
8
18 2 10 18 2

D1 8 8 20 8 8
V1 =
=
2 2 4
D
2 4
5 2 10 5 2

1 8 20 1 8

V1 =

[8 * ( 2) * ( 20) + 4 * ( 10) * ( 8) + 2 *18 * 8] [( 8) * ( 2) * 2 + 8 * ( 10) * 8 + ( 20) *18 * 4]


[2 * ( 2) * ( 20) + 4 * ( 10) *1 + 2 * 5 * 8] [1 * ( 2) * 2 + 8 * ( 10) * 2 + ( 20) * 5 * 4]

V1 =

2976
= 4.35V
684

-53-

V2 =

D2
=
D

2 2 8
2 8
5 18 10 5 18

1 8 20 1 8
684

8 2 4
2 4
5 2 18 5 2

D3 1 8 8 1 8
V3 =
=
684
D

Star Delta ( ) and Delta Star ( ) transformation :1. ) Delta Star ( ) transformation :If the value of RAB , RCA , RBC are
known, and we need to get the values
of RA , RB , RC ; then :RA =

RAB

RAB RCA
+ RCA + RBC

RB =

RAB RBC
RAB + RCA + RBC

RC =

RCA RBC
RAB + RCA + RBC

If RAb = RBC = RCA = R , in this case RA = RB = RC =


or R =

R
3

-54-

R
= R
3

30

10

30

10
C

10
C

30

2. ) Star Delta ( ) transformation :If the value of RA , RB , RC are known ,


and we need to get the values of RAB ,
RCA , RBC ; as follows :RAB =

RA RB + RB RC + RA RC
RC

RBC =

RA RB + RB RC + RA RC
RA

RCA =

RA RB + RB RC + RA RC
RB

If RA = RB = RC = R , in this case RAB = RCA = RBC = R = 3 RY


A

4
B

12

12

4
C

B
12

-55-

Examples of star and delta connections and transformation :-

Delta connection

Star connection

Example :- Find the current flow in the 25 V source for the following circuit :Solution :-

Ra =

5 *15
= 2.5
5 + 10 + 15

Rb =

10 * 15
= 5
5 + 10 + 15

Rc =

5 *10
= 1.67
5 + 10 + 15

RT = { (Ra + 10 ) // (Rb + 8) } + (Rc + 5)

RT = { (2.5 + 10 ) // (5 + 8) } + (1.67 + 5)
RT = { 12.5 // 13 } + 6.67

12.5 * 13
RT =
+ 6.67
12.5 + 13
RT = 6.37 + 6.67 = 13.04
I=

E
25
=
= 1.92 A
RT 13.04

25 V

-56-

Rc

Rb

Ra

10

Example :- For the following network , find I ?

Solution :- The resistances ( 6 , 3 , 3 ) are delta , can convert to star connection


as follows:

Ra =

6*3
6*3
= 1.5 , Rb =
= 1.5
6+3+3
6+3+3

RT = [(4 + R a ) // (2 + R b )] + R c

= [(5 .5 ) // (3 .5 )] + R c

5 .5 * 3 .5
=
+ 0 .75
5 .5 + 3 .5

I=

6
E
=
= 2.077 A
RT 2.889

-57-

, Rc =

3*3
= 0.75
6+3+3

Example :- Find I for the following cct. network :I


12.5

10
5

30

12 V
15

20

Solution :- The resistors ( 5 , 10 , 20 ) are star convert to delta


I

a
12.5

Rac

12 V

Rab

30

15
Rbc
b

Rab =

10 * 20 + 10 * 5 + 5 * 20
= 70
5

Rac =

10 * 20 + 10 * 5 + 5 * 20
= 17.5
20

Rbc =

10 * 20 + 10 * 5 + 5 * 20
= 35
10

It is clear that ( 12.5 // Rac ) and (15 // Rbc ) and (30 // Rab ) , hence the
circuit can be reduce to the following network :-

-58-

R1
R3

12 V
R2

R1 = (12.5 // Rac ) =

12.5 *17.5
= 7.3
12.5 + 17.5

R2 = (15 // Rbc ) =

15 * 35
= 10.5
15 + 35

R3 = (30 // Rab ) =

30 * 70
= 21
30 + 70

RT = (R1 + R2 ) // R3
= (7.3 + 10.5) // 21
= 17.8 // 21 =

I=

17.8 * 21
= 9.634
17.8 + 21

12
E
=
= 1.246 A
RT 9.634

-59-

Network Solution :To solve a circuit is to find the current and voltage in all branches.
1) Loop ( Mesh ) current method :Example( 1 ):- Find the current through the 10 resistor of the network
shown:

Solution : m Loop * + Loop * . ( = Loop )


The loop equations are :Loop 1 :- ( 8+3 )I1 + 3I2 +8I3 + 15 = 0
Loop 2 :- ( 3+5+2 )I2 + 3I1 +5I3 = 0
Loop 3 :- ( 10+8+5 )I3 + 8I1 +5I2 = 0
Rearrange the equations , then :-11I1 + 3I2 +8I3 = -15
3I1 - 10I2 +5I3 = 0
8I1 + 5I2 - 23I3 = 0

-60-

15
11 3
3 10 0

5
0
D3 8
=
= 1.22 A
I3 =
8
D 11 3
3 10
5

8
23
5
I 3 = I10 = 1.22 A

Example( 2 ):- Solve following circuit diagram;

Solution :-I1 ( 5+7 ) + 7I2 + 20 5 = 0


-I2 ( 7+2+6 ) + 7I1 + 6I3 + 5 + 5 + 5 = 0
-I3 ( 6+8 ) + 6I2 5 30 = 0
Rearrange;
-12I1 + 7I2 +0 = -15

-------------------

(1)

7I1 - 15I2 +6I3 = -15

-------------------

(2)

0 + 6I2 - 14I3 = 35

-------------------

(3)

-61-

0
15 7
15 15 6

14 2610
6
D1 35
I1 =
=
=
= 1.862 A
0 1402
D 12 7
7
15 6

0
14
6

I2 =

D2
=
D

12 15 0
7
15 6

0
35 14

1402

1470
= 1.049 A
1402

15
12 7
7
15 15

0
6
35 2875
D
I3 = 3 =
=
= 2.05 A
D
1402
1402

Example( 3 ):- Find the current in the 10V source , for the following network;
4

10V

I1

I2
5A

Solution :I2 = -5 A
Hence , we need only one equation to solve this circuit
-I1 ( 4+6 ) + 6 * ( -5 ) + 10 = 0
-10I1 20 = 0 -10I1 = 20
I1 =

20
= 2 A
10

-62-

Example( 4 ):- Solve the following circuit diagram, also find the voltage across
15 resistance?

5O

18V

I1
6O
12V

4O

I2

3O

7O

8O

I3

2O

9O

15O

I4

2A

Solution:I4 = 2 A
-I1 ( 4+6+5 ) + 4I2 + 12 18 = 0
-I2 ( 8+3+4+7 ) + 4I1 + 8I3 - 12 = 0
-I3 ( 15+2+8+9 ) + 8I2 + 15 * 2 = 0
Rearrange:-15I1 + 4I2 +0 = 6

-------------------

(1)

4I1 - 22I2 +8I3 = 12

-------------------

(2)

0 + 8I2 - 34I3 = -30

-------------------

(3)

-63-

I1 =

D1
D

I2 =

D2
D

I3 =

D3
D

V15 = I15 * R15


= ( I3 I4 ) * 15
= ( I3 2 ) * 15
Example( 5 ):- Solve the following circuit diagram .
10A

20A

60

15

5A

40

Solution:- The above diagram can be reduced to the following diagram;

-64-

50V

I1

60

15

I2
60V

75V
40

-I1 ( 5+15+60 ) + 60I2 - 50 75 = 0


-I2 ( 3+60+40 ) + 60I1 + 60 = 0
Rearrange:-

I1 =

I2 =

-80I1 + 60I2 = 125

-------------------

(1)

60I1 - 103I2 = -60

-------------------

(2)

D1
D

D2
D

Example( 6 ):- Solve the following circuit diagram:


10

6
2

I2

I1
20V

Vo

6A

-65-

Solution:-( 6+2 )I1 + 2I2 +20 - Vo = 0

-------------------

(1)

-( 10+2+4 )I2 + 2I1 + Vo = 0

-------------------

(2)

I2 - I 1 = 0

-------------------

(3)

-8I1 + 2I2 +20 -16 I2 + 2I1 = 0

-------------------

(1)

-6I1 - 14I2 = -20

-------------------

(2)

Add eq. 1 & eq. 2

From eq. 3
I1 I2 = -6
20 14

D1 6 1 20 84
I1 =
=
= 3.2 A
=
D
6 + 14
6 14
1
1

6 20

6 36 + 20
D2 1
I2 =
=
= 2.8 A
=
D
20
20

-66-

Example ( 7 ) : Solve the following circuit , using loop current method :6

12V

Ib

Vo

3A

Ia

Ic

Solution:
Loop a :)(1

-------------------

)(2

-------------------

)(3

-------------------

-( 8+7+9 )Ic + 8Ia + 7Ib = 0

)(4

-------------------

Ib I a = 3

-( 5+8 )Ia + 8Ic - Vo = 0


Loop b :-( 6+7 )Ib + 7Ic + Vo 12 = 0
Loop c :-

-: Vo
Vo .
Vo )
( Vo Vo
.
-67-

Loop a+b :
-------------------

( 1 )

-24Ic + 8Ia + 7Ib = 0

-------------------

( 2 )

Ib I a = 3

-------------------

( 3 )

-13Ia - 13Ib + 15Ic = 12

-------------------

( 1 )

8Ia + 7Ib -24Ic = 0

-------------------

( 2 )

Ia I b

-------------------

( 3 )

-13Ia - 13Ib + 15Ic 12 = 0


Loop c :-

Rearrange Eq.s :-

=3

12 13 15
0
7
24

0
1
3
D
2610
Ia = 1 =
=
= 1.862 A
D 13 13 15 1402
8
7
24

1
1
3

Ib =

D2
D

Ic =

D3
D

-68-

Example, (Sheet 4 Q. 25): Solve the following circuit diagram using loop
current:
12

15

9V

13V

30

6V

1.2A

0.8A

20

Solution:15 // 30 =

15 * 30
= 10
15 + 30

12 + 3 = 15
15

9V

13V

Ia

6V

Ib

10

1.2A

Ic
20
16V

Loop a:-( 15+7 )Ia + 6 + 9 + 7Ib - 13 = 0

-69-

-------------------

(1)

Loop b:-------------------

(2)

-20Ic 16 6 + Vo = 0

-------------------

(3)

Ic Ib = 1.2

-------------------

(4)

-( 7+10 )Ib - Vo + 13 + 7Ia = 0


Loop c:-

Loop b+c:
-17Ib - 20Ic + 7Ia 9 = 0
Rearrange Eq.s :Loop a:

-22Ia + 7Ib = -2

-------------------

(1)

Loop b+c:

7Ia - 17Ib -20Ic = 9

-------------------

(2)

Ib Ic = -1.2

-------------------

(3)

Example, (Sheet 4 Q. 7): Solve the following circuit diagram:

-70-

Solution:
120

10

Ia

20V

8V

80

1
Ib

10V

4V

10V

Loop a :-( 10+120+2+80 )Ia + 80Ib - 8 + 20 = 0

-------------------

(1)

-------------------

(2)

Loop b :-( 2+5+80+1 )Ib + 80Ia - 4 - 10 + 10 = 0


Rearrange Eq.s :-212Ia + 80Ib = -12

-------------------

(1)

80Ia - 88Ib = 4

-------------------

(2)

-71-

Nodal voltage:Example 1 :- Solve the following circuit using the nodal voltage method:

VC - VA
E1

R1
I1

VB - VC

VA - VB
R5

R6

I5 B I6

I2
VB

R3

R2

R4

VC

I3
VA

E2

I4

Solution :

Choose reference point

Let D be a reference point


Kcl at B:
I5 I 2 I 6 = 0

(VA VB ) G5 ( VB E2 ) G2 ( VB VC ) G6 = 0
Kcl at A:
I3 I 5 I 1 = 0
-VA G3 ( VA VB ) G5 [( VA VC )- E1] G1 = 0

-72-

N=4
IN = N-1 = 3

Kcl at C:
I 6 + I1 I 4 = 0
( VB VC ) G6 + [( VA VC )- E1] G1 VC G4 = 0
Rearrange:
A:

( VB VA ) G5 -VA G3 + ( VC VA ) G1 - E1G1 = 0

B:

(VA VB ) G5 + ( VC VB ) G6 - VB G2 + E2 G2 = 0

C:

( VB VC ) G6 VC G4 + ( VA VC ) G1 - E1G1 = 0

Hence, we can arrange the above equations in the following form:A:

- VA ( G1 + G3 + G5 ) + VBG5 + VCG1 + E1G1 = 0

-----------

(1)

B:

- VB ( G2 + G5 + G6 ) + VAG5 + VCG6 + E2G2 = 0

-----------

(2)

C:

- VC ( G1 + G4 + G6 ) + VAG1 + VBG6 - E1G1 = 0

-----------

(3)

Then, we can find VA , VB , VC by the determinate method .


Example 2 :- Solve the following circuit diagram using nodal voltage .

Solution:
First we simplify the circuit and make a less nodal point.

-73-

N = 4 ; IN = 4 1 = 3
Let D be a reference point
A:

1 1 1
1
1 1 30 35
VA + + + VB + VC + +
+
=0
19 8 7
7
8 19 8 19

B:

1 1
1 51
VB + + VA + 3 = 0
7 33
7 33

C:

1 1 1
1 1 30 35
VC + + + VA +
=0
6 8 19
19 8 8 19

Rearrange
-0.321 VA + 0.143 VB + 0.178 VC = -5.592

-----------

(1)

0.143 VA - 0.174 VB = 1.455

-----------

(2)

0.178 VA - 0.344 VC = 5.592

-----------

(3)

VA , VB , VC

-74-

Example 3 :- Solve the following circuit using nodal voltage method:


3

30
C
15

50

15V
20

Solution:
Let D reference
VA = 15 V

B:

1 15
1
1
1 1 1
VB + +
+ + + VC + VE = 0
20
30
6 9 30 20 6

C:

1
1
1 15
1 1 1
VC + +
+ + VB + + VE = 0
50
30 3
30 3 50 15

D:

1 1
1
1
1
VE +
+ + VB + VC = 0
50
20
20 50 5

Then rearrange the above equations and find VB , VC , VE .

-75-

Example, (Sheet 4 Q. 24): For the following circuit diagram, find I & I1,
using nodal voltage method:
12V
20

15

I1

25
25
0.3A

17V
15V
B
15

35

0.8A

Solution:
First; let D reference:V
1
1
1 V
12 15
+
+ 0.3 = 0
+
+ + B + C +
40 25 20 25 40 40 25

-----------

(1)

1
1
1 V
15
I =0
+ + + A
25 15 35 25 25

-----------

(2)

C : VC

1 12 V A
+
+ 0.8 + I = 0

40 40 40

-----------

(3)

VB + 17 = VC

-----------

(4)

A : V A

B : VB

Then the above equations can be minimized to:B + C:


1
1 15
1 12
1
1
1
+ 0.8 (VB + 17 ) = 0
+ VA +
VB + +
40
25 40 40
25 15 35 25

-76-

-----------

(1)

A:

(V + 17 ) + 12 + 15 + 0.3 = 0
1
1 V
1
VA +
+ + B + B
40
40 25
40 25 20 25

-----------

(2)

Then solve to find VA & VB; hence we can find VC from eq. ( 4 ); to find I sub.
VA & VB in eq. ( 2 ); and to find I1; then I1 = I + 0.8
Second solution; let B reference
Hence VC = 17 V
1
1
1 12 17 VD 15
+
+
+
+ 0.3 = 0
+
+ +
40 25 20 40 40 20 25

-----------

(1)

1
1
1 V
+
+ + A 0.3 0.8 = 0
15 35 20 20

-----------

(2)

A : VA

D : VD

Then find VA & VD from the above equations; hence to find I;


C:

17 VA
12
+
+ 0.8
+I =0
40 40
40

And to find I1; I1 = I + 0.8

-77-

:(9)
.

/
Network Theorems:1- Superposition Theorem:In any circuit network contain more than one sources ( voltage or
current ) to find the current ( or voltage ) in a certain part of a network , remove
the sources of the network and find the current ( or voltage ) in the existence of
only one source each time. The resultant current ( or voltage ) will be the
algebraic sum of current ( or voltage ) due to all sources when acting
independently once a time .
(Removing the sources means:- Short circuiting the voltage source and open
circuiting the current source) .
Example 1:- In the following circuit diagram, find all branch current's using
superposition theorem:25V

I3

I4

I1

3
I2

I5
4

6
3A

-79-

Solution :1.) Effect of 25 V source :-

I1 =

25
= 2.5 A
7+3

I 2 =

25
= 2.5 A
4+6

I 3 = I1 + I 2 = 5 A

2.) Effect of 3 A source :I"3

I1 = 3 *

7
= 2.1A
7+3

I 4 = 3 *

3
= 0.9 A
7+3

I 2 = 3 *

4
= 1.2 A
4+6

6
I 5 = 3 *
= 1.8 A
4+6

I"4

I"1

6
3A

I"5
I 3 = I1 I 2 = I 5 I 4 = 2.1 1.2 = 0.9 A

3.) Superpose :I1 = I1 + I1 = 2.5 + 2.1 = 4.6 A

I 2 = I 2 I 2 = 1.2 2.5 = 1.3 A

I 5 = I 2 + I 5 = 2.5 + 1.8 = 4.3 A


I 4 = I1 I 4 = 2.5 0.9 = 1.6 A

I 3 = I 3 + I 3 = 5 + 0.9 = 5.9 A

-80-

I"2

Example 2:- For the following circuit network, find the current in all branches,
using superposition theorem:I3
I2
30

40
60

I1

270V

150V

Solution:1.) Effect of 150 V source:I'3


I'2
30

40
I'1

60

150V

RT = 40 +
I1 =

60 * 30
= 60
60 + 30

150
= 2.5 A
60

I 2 = 2.5 *

30
= 0.83 A
60 + 30

I 3 = 2.5 *

60
= 1.67 A
60 + 30

-81-

2.) Effect of 270 V source:-

RT = 30 +
I1 =

40 * 60
= 54
40 + 60

270
= 5A
54

I 2 = 5 *

40
= 2A
40 + 60

I 3 = 5 *

60
= 3A
40 + 60

3.) Superpose :I1 = I1 I 3 = 2.5 3 = 0.5 A


I 2 = I 2 + I 2 = 0.83 + 2 = 2.83 A

I 3 = I 3 I1 = 1.67 5 = 3.33 A

Example 3:- Find the current in all branch in the following circuit diagram:2A

15V

I1

I2
I4

I3

I5
12V

-82-

Solution:1.) Effect of 15 V source:-

I1 =

15
= 2.12 A
5*8
4+
5+8

I 2 = 2.12 *

8
= 1.3 A
5+8

I 3 = 2.12 *

5
= 0.82 A
5+8

2.) Effect of 12 V source:5

I"3
I"2
8

I"1
12V
I1 =

12
= 1.57 A
8*4
5+
8+4

I 2 = 1.57 *

4
= 0.52 A
8+4

I 3 = 1.57 *
= 1.04 A
8+4

-83-

3.) Effect of 2 A source:2A


Vo
I'"2

I'"4

I'"5

I'"3

1
1 1 1
= + + RT = 1.74
RT 8 4 5
Vo = 2 * RT = 3.5V
I1=

Vo
= 0.88 A
4

I 2 =

Vo
= 0.7 A
5

I 3 =

Vo
= 0.44 A
8

I 4 = 2 I1= 1.12 A

I 5 = 2 I 2 = 1.3 A

3.) Superpose :I1 = I1 I 3 I 4 = 2.12 1.04 1.12 = 0.04 A


I 2 = I 2 I1 I 2 = 1.3 1.57 0.7 = 0.97 A

I 3 = I 2 I1 + I 5 = 1.3 1.57 + 1.3 = 1.03 A


I 4 = I 3 + I 2 I 3 = 0.82 + 0.52 0.44 = 0.9 A
I 5 = I1 I 3 + I1= 2.12 1.04 + 0.88 = 1.96 A

-84-

4
I'"1

:(10)
.

/
2-) Thevenin's Theorems:.
Any two terminal linear network can be replaced by an equivalent circuit
of a voltage source ( Eth ) and a series resistor ( Rth ); as shown in figure below:-

Hence; I =

Eth
Rth + RL

Steps to find Eth & Rth :1. Remove that portion of the network across which the Thevenins
equivalent circuit is to be find.
2. Mark the terminals of the remaining two terminal network.
3. Calculate Rth by first setting all sources to zero ( voltage sources are
replaced by short circuits and current sources are replaced by open
circuit ), and finding the resultant resistance between the two marked
terminals.
4. Calculate Eth by first returning all sources to their origin positions and
finding the open circuit voltage between the marked terminals.
5. Draw the Thevenins equivalent circuit with the portion of the circuit
previously removed replaced between the terminals of the equivalent
circuit.

-85-

Example 1:- For the following circuit diagram, find the current in ( 6 )
resistor?
3

2
4

7A

7A

25V

Solution:3

2
4
25V

1.) Find Rth :


3

B
Rth

Rth = {(2 + 3) // 4} + 5
20
5*4
=
+ 5 = 7.22
+5 =
9
5 + 4

-86-

2.) Find Eth :

25
A
9

Voc V1 + V2 = 0
25
Voc 4 * + (7 * 5) = 0
9

100
35 = 23.89V
Voc =
9
Eth = 23.89V

Rth = 7.22
6
Eth = 23.89 V
B

I=

Eth
Rth + RL

23.89
= 1.8 A
7.22 + 6

-87-

Example 2:- Find the current in the 25 resistor for the following circuit
network?

40

10

2V

25

20

20

Solution:1.) Find Rth :


Rth = (10 // 20 ) + (40 // 20 )

40

10
A

10 * 20 40 * 20
Rth =
+
= 20
10 + 20 40 + 20

20

20

2.) Find Eth :

2
2

Voc + 10 * 40 * = 0
30
60

80 20 40

=
= 0.67V
Voc =
60 30 60

10

2
A
60
40

2
A
30

2V

B
Voc

Eth = 0.67V

20

-88-

20

I=

Eth
Rth + RL

I =

0.67
0.67
=
A
20 + 25
45

Example 3:- Find I in the ( 9 ) resistor for the following cct. diagram?

6A

10

8
4A

25V

Solution :-

Rth = 7 + 10 + 8 = 25

-89-

6A

4A

Voc

10

6A

25V

Voc (4 * 7 ) (10 * 10 ) 25 (8 * 6 ) = 0
Voc = 201V

4A

10A

I =

Eth
Rth + RL

201
= 5.91 A
25 + 9

-90-

Norton's Theorems:Any two terminal linear network can be replaced by an equivalent circuit
consisting of a current source and a parallel resistor.

E th
Rth

RN = Rth as before .
IN = Isc = short circuit current between the two terminals of the active network.
Example 1:- Find the current in 25 resistor for the following circuit network
using Norton's Theorem?

2V

40

10
25

20

-91-

20

Solution:First find RN :RN = (10 // 20 ) + (40 // 20 )


=

40

10
A

10 * 20 40 * 20
+
= 20
10 + 20 40 + 20

20

20

Second find IN :-

I=

2
10 * 40 20 * 20
+
10 + 40 20 + 20

2
1
= A
8 + 10 9
I

1 20
1 20
I3 = * & I4 = *
9 40
9 40

2V

B
IN

KCL at A
I1 I N I 3 = 0
IN = I1 I 3
1 40 1 20
= * * = 0.033 A
9 50 9 40

I L = 0.033 *

40

10

20

I2

I1

1 40
1 10
I1 = * & I 2 = *
9 50
9 50

20
= 0.0147 A
20 + 25

-92-

20
I3

I4

Example 2:- Find I in 50v voltage source, for the following circuit using
Norton's Theorem?
25

12

17

30

20

65V

50V
45V

Solution:1.) Find RN :25

12

17
A

30

20

R1

R2
A

30
B

-93-

R3

20

R1 =

17 * 25
12 * 17
= 7.8 , R2 =
= 3.78
54
54

, R3 =

25 *12
= 5.56
54

RN = [(R1 + 30 ) // (R3 + 20 )] + R2
= [37.8 // 25.56] + 3.78 = 19

2.) Find IN :25

Ic
12

17
A

30
Ia
65V

IN
B

20
Ib
45V

-47Ia + 17Ic + 65 = 0
-32Ib + 12Ic - 45 = 0
-54Ic + 17Ia + 12Ib = 0
After find Ia , Ib , Ic
IN = Ia I b

I N - Ia - IL = 0
IL = I N - Ia = I N

50
50
= IN
19
RN

-94-

Maximum Power Transfer:A load will receive maximum power from a d.c. network when its total
resistive value is exactly equal to the Thevenin resistance of the network.
For Thevenin cct.

Nortan cct.

IN

RN

RL

For Max. power


RL = Rth

RL = RN
2

PLmax .

Eth
* RL
= I L2 RL =
+
R
R
th
L

PLmax .

RN
* RL
= I RL = I N
RN + RL
2
L

Eth2
=
* Rth
4 Rth2
PLmax . =

= I N2

Eth2
4 Rth

PLmax . =

EN2
* RN
4 RN2

I N2 RN
4

Under Max. Power transfer conditions, the efficiency is:% =

Po
* 100%
Pi
=

VL I L
V
*100% = L * 100%
Eth I L
Eth

-95-

Eth = VL + Rth Ith


( for max. power transfer )

Q Rth = RL

Eth = VL + RL IL
= VL + VL = 2VL

VL
V
* 100% = L * 100% = 50%
2VL
Eth

The efficiency will always be 50% under max. power transfer conditions .
* Practical example:I
Rth

PL = I 2 RL

RL
Eth

E
* RL
=
Rth + RL

Let

Rth = 3

&

RL = 1 &

Eth = 15 V

15
PL =
*1 14W
3 +1
2

For RL = 2

15
PL = * 2 = 18W
5

For RL = 3

15
PL = * 3 = 18.75W
6

-96-

For RL = 4

15
PL = * 4 = 18.36W
7

For RL = 5

15
PL = * 5 = 17.57W
8

Note that when RL = Rth , we


get the max. power of PL .
1
2

1
2

Hence PL = Pin = EI
or Pin = 2 PL
Example 1:- Find the value of RL for maximum power transfer to RL , and
determine the power delivered under these conditions ?

Solution:First remove RL , and find the equivalent resistance ( Rth )

-97-

Rth = (3 // 6 ) + 12
Rth =

3*6
+ 12 = 14
3+6

For Max. power RL = Rth


RL = 14

Eth = Vab =

12

18 * 6
= 12V
6+3

6
Vab

E2
(12 ) = 2.57W
= th =
4 Rth 4 *14
2

Pmax .

18V
b

Example 2:- Find the value of RL for the following cct. for max. power transfer,
and find PL?

Solution:8

4
a

Rth
b

-98-

Req . = Rth = (8 // 7 ) // (6 + 4 )
=

15 * 10
= 6 = RL
15 + 10

Voc = Vab = Eth =

120
* (8 + 7 )
(6 + 4 + 8 + 7 )

120
* 15 = 72V
25

72
PL = I RL =
* 6 = 216W
(6 + 6)
2

or
Eth2
72
PL =
=
= 216W
4 Rth 4 * 6

Rth = 6

RL= 6

Eth = 72 V
b

-99-

Example 3 (sheet 5, fig. 20):- Find the maximum power in ( R ), for the
following cct. diagram?
1

6V

1
10V

2A

6
6V

Solution:1-) Find Rth.:-

-100-

Req. = Rth = 4 + 1 + 3 + 2 = 10 = R

2.) Find Eth :-

-101-

IX

6V

10V

2A

4
2A

IY

Voc
6

6V

IZ

4 I x 10 + (1 * 2 ) = 0 I x = 2 A
12 I y + 10 6 + (2 * 6 ) = 0 I y = 1.33 A
8 I z + 6 + (2 * 2 ) = 0 I z = 1.25 A

From KVL
6 Voc + (4 *1.25) + (6 *1.33) (2 * 2 ) = 0
Voc = 6 + 5 + 8 4 = 15V

I=

15
= 0.75 A
10 + 10
-102-

Pmax . = I 2 R

Rth = 10

= (0.75 ) *10 = 5.625W


2

RL= 10

Eth = 15 V
B

Example 4 (sheet 5, fig. 21):- Find the maximum power in ( R ), for the
following cct. diagram?
0.86V
12

R
15

18

80mA

20mA

5
0.3V

Solution:First find Req.:-

Req. = (12 + 18) // (15 + 5) //8 = 4.8

-103-

Second find IN:0.86V


12
A

Isc

0.36V

15
B

18

1.2V
5
0.3V

0.5V
30
Isc
IX

20

B
IY
1.5V

30 I x + 0.5 = 0 I x =

0.5
30

20 I y + 1.5 = 0 I y =

1.5
20

I N = I sc = I y I x =

1.5 0.5

= 58.3mA
20 30
-104-

58.3
A
2

Pmax .

58.3

*10 3 * 4.8 = 4.1mW


=
2

-105-

You might also like